Tuesday, August 25, 2020

Meditation in the Buddhist Traditions-Free-Samples-Myassignment

Question: What are the Nature of Meditation in the Buddhist conventions and why it is Important? Answer: Subject Definition 2500 years back in India Buddhism started and work date in the East it is the most predominant religion. Around the world, Buddhism has more than 360 million adherents. In USA exclusively one can discover more than one million Buddhist supporters. The idea of Buddhism has affected even the western societies of the world through its lessons in particular territories, for example, in reflection rehearses. Buddhism advances the conviction of peacefulness. In 500 BCE in India there was an Indian cost and his name was Gautama. The youthful cost was in interior disturbance to see the sufferings of the life beyond the royal residence he lived in. He went out to look for answer to numerous inquiries he had about existence. He prevailing with regards to finding the appropriate responses and lectured his lessons f or a mind-blowing remainder (Gethin, 2015). Buddhist Schools There are numerous types of Buddhism. A few devotees have faith in divinity reveres while others accept on customs. There are numerous ways of thinking however the principle lessons of all the various ways of thinking stays to be the equivalent and that is finishing lifes sufferings and completion the resurrection pattern of life. After Gautama Buddha kicked the bucket eighteen distinct schools of Buddhism developed. Every one of them were various portrayals of Buddhist lessons. Gradually with time all these various schools were converged into two elementary schools-Theravada and Mahayana (K and Thera, 2005). Theravada school according to this school of Buddhist encouraging the possibilities inside an individual to achieve Buddha hood can be released distinctly through exertion and difficult work. Mahayana school The Mahayana Buddhism considered each individual have the potential inside themselves to accomplish Buddha hood. Subsequently they should look for towards salvation utilizing Superior creatures known as Bodhisattvas. Reflection care Individuals are regularly having the misguided judgment that an otherworldly life is one which exists some place in the sky. Our day by day life is simply ordinary indications of our own selves. Numerous individuals additionally believe that to be otherworldly one needs to leave their ordinary Earthly life and be separated from everyone else some place away from loved ones. There can't be much else wrong than this idea. To be profound intend to be thoughtful and to be an unassuming person, to be benevolent to ones own self and to others too. The inquiry is how might one develop graciousness in their heart? Trying to say oneself to be kinder doesn't generally turn out to be appropriately. For building up this sort of a manner of thinking contemplation is suggested. This is otherwise called profound thought in Buddhism. The lessons ask individuals that when they wake up they should begin a profound thought propensity. Such a contemplation practice must be done before leaving the bed wh en one has recently woke up and the psyche is vacant of any common musings (Chodron, 2013). Writing Review Contemplation Contemplation is a game-plan of the psyche and the body which is utilized by a person for isolating themselves from their emotions and musings and discharging themselves to build mindfulness. Contemplation not really has a strict component related with it. It is a human action which now and again is utilized as a treatment for guaranteeing great wellbeing and improving the human insusceptible framework. An individual who effectively ponders won't think or judge everything which goes over. They will simply be in harmony and mindfulness and will trust in embracing current circumstances. Reflection is a fundamental piece of Buddhism. In any case, one who is contemplating in Buddhism isn't attempting to contact the Divine. In Buddhism a human has duality inside them as they have a brain and a body. Joining these two components for increasing everlasting genuine feelings of serenity is contemplation (Anonymous, 2009). Speculations of Meditation Techniques Researched All the various types of reflection require the person to sit straight up erect. In such a stance the best outcomes are adored to be accomplished. Various speculations of reflection helps individuals in relieving changed sort of issues. The point of every one of these hypotheses are anyway the equivalent. The accomplishment of harmony and interior bliss is the fundamental point of all reflection styles. Zazen or Zen Meditation The term Zazen implies a type of situated zen or reflection in the sitting structure. This type of reflection practice begins once more from the Chinese type of Zen Buddhism. Indian priest named Bodhidharma from sixth century CE can be the followed to be the wellspring of this type of Buddhism. By and large, in this type of contemplation the individual sits over a tangle or a pad on the floor by intersection the legs. Additionally it was customarily called as the lotus or the half lotus act. Today individuals likewise practice this on seat. The most noteworthy component of this stance is sitting straight so the back is totally straight (Shaw, 2016). Eyes are kept brought down and mouth shut and one glances at the ground a few feet separation on the floor. Vipassana-This type of reflection originates from Pali and the word Vipassana implies clear vision. This is another ordinary Buddhist work on originating from the Theravada school of Buddhist customs. Vipassana reflection includes two phases. The primary stage is the place an individual ponders and attempts to balance out the psyche on one point. This makes extensive enhancement for the fixation level of the person. In the second stage the contemplating individual unmistakably observes with shut eyes each sense in each piece of the body. It seems as though the psyche is centering starting with one cell of the body then onto the next without laying on a specific point (Chen, 2014). Care Meditation This is a blend of the regular contemplation practices of Buddhism and different styles, for example, Vietnamese Buddhism Zen reflection rehearses. Care while taking in and out during reflection is the essential center region of this kind of contemplation practice. This reflection is likewise a piece of the Vipassana style of contemplation in Buddhist customs. The essential center zone is right now and not being critical to musings, sentiments and faculties structures significant components of care reflection (Neale, 2006). Metta or the Loving Kindness Meditation Metta implies love, consideration, modesty, generosity and kindheartedness in Pali language. The Tibetan and the Theravada school of Buddhist customs began this type of contemplation style. Here the principle center is to build the motion of understanding others, positive enthusiastic turn of events, improvement of empathy including positive and cherishing way to deal with ones own self too (Liveware, 2015). Advantages of Meditation There are unlimited advantages to intervention rehearses. With the end goal of improved understanding the extensive rundown is bifurcated into physiological advantages, mental advantages and otherworldly advantages (Horowitz, 2016). There has been a great deal of research about the different favorable circumstances reflection causes to the human psyche and body. Despite the fact that this is an antiquated type of training today it is quick turning into a logical procedure which demonstrates that with care while contemplating individuals can make huge effect even on their minds only the way that can on their bodies (Corliss, 2014). Expanded degrees of fixation Meditation can successfully improve the degree of mindfulness, fixation or center in people. Among all the strategies utilized for improving fixation reflection is the best one. Contemplation affects improving the vitality level of the body. While contemplating the body and psyche gets associated with the vitality power source which thusly enables the individual to do a great deal of performing multiple tasks with a similar core interest. Utilizing sacred globules while contemplating is one extraordinary strategy which aides killing all blockages which happens in the vitality level of human starts. Fixation levels are improved with serious contemplation as well as it is supported, upgraded, engaged and all around coordinated. A characteristic pressure buster Complexities of current life has made everybody pushed. Such pressure is in certainty expanding at a quick rate. It can prompt numerous medical issues and influence individuals truly. Stress can cause mental and physical disease both. Reflection has been seen to have a lot of mental impact. It affects the pressure, despondencies, nervousness, anxiety and agony of the human body. With ordinary reflection practice one can turn out to be sincerely balanced out. Negative considerations can be kept far as reflection encourages the human cerebrum to concentrate on positives as it were. To decrease worry there can't be any preferable arrangement over contemplation Helps get out from under away from unfortunate propensities Fastness of the cutting edge life causes individuals to create numerous undesirable propensities every day. Eating low quality nourishment is one such model. Individuals know its wellbeing impacts yet at the same time don't have the conviction to change their unfortunate propensities. By rehearsing contemplation one can stop such unfortunate propensities without any problem. This is on the grounds that with ordinary reflection practice it is accepted that one will have the option to isolate oneself from the enthusiastic connections related with the act of any undesirable propensity. This feeling of opportunity is sublime and will assist one with being resolved to split away from each sort of unfortunate propensity one has sustained since years. Helps improving dynamic and basic reasoning The intensity of the human mind can be upgraded with contemplation. Various research examines exhibit that individuals who are occupied with rehearsing careful reflection every day will in general be having better recollections. They appear to be thinking far superior to other people. The intellectual working of the human mind is made increasingly proficient through reflection. Contemplation centers around better breathing procedures which empowers more measure of unadulterated oxygen to go inside the human framework. Subsequently the mind performs much better. Along these lines, while disappointing circumstances come cross the mind stays cool and takes the best reasonable choices. Reflection improves communication with the self-One can a

Saturday, August 22, 2020

Understanding Essay Titles

Understanding Essay Titles Understanding Essay Titles Today, it is uncommon that you would be given instant exposition titles. Typically what you get is a paper question or a brief. The primary fixing in an effective article is understanding what you have to expound on. Most article questions contain four segments: viewpoint, center, guidance and subject. This recipe works the two different ways, possibly you get an inquiry which as of now contains these components, or you get a section of guidelines and you need to concoct an exposition title, in light of them. Other than your theme, the title ought to pass on the point of your contention, setting and what you have to do. In this post, you will get some answers concerning the kinds of paper titles and what you ought to do with them. In light of the instructional action word, we can recognize 9 kinds of titles. Dissect You need to process related materials to discover key proof and significant components which impact the result. This implies you need to look at each snippet of data you find and give the crowd realities, rather than suppositions. Model: The Importance of Higher Education Look into These two regularly come in couple, since clearly you will contrast two focuses or components with find both their similitudes and contrasts. You can likewise clarify the noteworthiness of the coordinating or contradicting highlights you have found. Model: Batman VS Superman: Do Only Gods Have Superpowers? Portray This one is likely the most straightforward to comprehend, as you just need to call attention to the situation. No sentiments or clarifications are required. Your exposition will be founded on one of the five detects: what I see, what I smell, what I hear, what I taste, what I feel Model: A Day In the Life of a Blind Person Talk about On the off chance that you get a conversation sort of article title, be set up to pick a side. Distinguish the perspectives regarding the matter, take one and contend possibly in support of it utilizing realities, models and contradicting sees. Model: Photo Realism: Is It Necessary? Assess Put on the teachers’ shoes for a day. On the off chance that you are approached to assess something, you should dissect the topic and recognize its convenience or irrelevance. Model: The Issue of Using Electronic Device on the Road Inspect Here, you should take the subject under a magnifying instrument and investigate the littlest subtleties. It might be applied to individuals, occasions, various wonders, and different components. Model: Examine the Methods of Preventing Water Pollution Clarify This sort of paper title for the most part starts with a how or a why. It implies that the exposition will either give guidelines or uncover an issue and clarify why something is occurring the manner in which it does. Model: Obesity in the USA Legitimize These themes are normally disputable over the span of a worldwide conversation. You should discover clear proof to help the contention, clarify why the wonder that you are protecting has a spot to be. Model: Legalizing Abortions for Rape Victims All things considered, our administration is consistently accessible online to furnish understudies with custom article composing help at a sensible expense.

Research Paper Example | Topics and Well Written Essays - 1500 words

Research Paper Example 7). The post auxiliary instruction offers non-degree programs prompting endorsements, recognitions and degree levels. The framework doesn't have a second or higher doctorate, yet it offers post doctorate inquire about projects. As a result of this complexities and levels, the framework has been named as an out of date framework that requires advancement. Argumentative issues exist on the present association of levels in the training structure, the K-12 structure. Most American students particularly in writing and number juggling have blamed the training structure for deficiencies that that lead to scholarly inability. The main demonstrators of the presence of scholastic structure issues lay on inadequacy in work places, just as the heightening quantities of dropouts. This shows educators don't use proper systems to improve information perception and maintenance, and methods that invigorate learners’ enthusiasm for scholastics. This thesis examines scholarly changes, the differ ent features that it has, and recommended alterations. Pertinent instructional material and qualified educators contain important instructive parts for training at home or school. This empowers understudies to learn and pick up aptitudes and information. They likewise add to the capacity of understudies to go after financial security through employment rivalry. Satisfactory training gives understudies the instruments required for commitment in fitting municipal grown-up life (Rowan, and Miller, p. 5). The outcomes of coming up short on the instructive offices and segments have prompted decrease in the principles of training. The abberations and deficiencies of instructional material, educators and offices, show a fundamental and profound imperfection in the national instructive framework. These defects incorporate garbled and divided way to deal with national policymaking. They likewise demonstrate a defective arrangement of school fund based biased circulation of assets. Changed pe ople recommend varying answers for the issue of scholarly rebuilding in America. Furthermore, there have been countless changes of the present structure of American training since initiation. The issue lay with characterizing the ideal arrangement of sorting out and conveying of information in scholastic organizations. This outcomes to a quandary as a result of changed belief systems on instructive frameworks from reformists, just as researchers. What's more, lawmakers add to the misconception that encompasses the meaning of a perfect structure of training worth actualizing. Most lawmakers voice what they see as the expectation of voters (Futrell, p. 9). They do not have a comprehension of the truth of the insufficiencies in the instructive frameworks, and the necessities for instructive changes. This raises banter on the suitability of their belief systems (McClure, Wiener, Roza, and Hill, p. 10). There exists an all inclusive conviction in the worldwide network that instruction gi ves a basic asset that vital for including in life’s triumphs. Instruction gives the steadiness to gain abilities and the assurance to accomplish targets and accomplish exclusive requirements. It grants people to fathom aptitudes of life and the benefit of having such expertise; thus the need to find out its achievement. Subsequently, making a serious, instructive structure essentially adds to the prosperity of

Friday, August 21, 2020

Morality in war Essay Example for Free

Profound quality in war Essay Is war ever good? I feel that War is a vital piece of life. Some of the time power is simply the best way to safeguard or others. We are generally equivalents, and It is unethical to end the life of anybody, yet it is likewise indecent to leave the life of anybody alone taken. We have an obligation to help each other on the grounds that participation is the main way humankind can endure; we likewise reserve an option to shield ourselves. This implies on the off chance that another person is compromising you or anothers life, and you are equipped for helping, you have a commitment to ensure yourself or them. Deadly power will never be good, however imagine a scenario in which it’s the best way to ensure somebody. Let’s state that an assailant has broken into your home and is holding a firearm to you and your family. You likewise have a firearm pointed at the attacker. In this theoretical circumstance we should expect that the best way to spare your family is to execute the aggressor. You should settle on a decision to murder the assailant or let the aggressor execute you and your family. The two choices are shameless, so one needs to inquire as to whether its increasingly indecent to slaughter the assailant, or to let the aggressor execute your family? The undeniable decision to me is to slaughter the aggressor. I feel this is the correct decision on account of two reasons. One, our obligation to ensure ourselves and our family is bigger than the duty to not do damage to another. The subsequent explanation has to do with the extent of harm. The aggressor would accomplish more damage in murdering my family and me then I would do in executing him. This model permits us to see, from a more minor perspective, when deadly power is important. Things get progressively convoluted when we take a gander at whole nations instead of only one family, however my view point continues as before. Deadly power is possibly allowed when it is simply the main choice to protect or others. War is essential since individuals once in a while settle on corrupt choices that set others in a place where there is no other choice, yet is war ever good? Murdering is constantly indecent, and executing is a piece of war, so parts of war are unethical, however does this mean war is naturally improper? At times it is the least improper decision; this settles on it the most good decision, so it is at some point moral. I accept that a considerable lot of the wars we have had have been shameless. I don’t imagine that the United States has been supported in it’s activities without fail. â€Å"It is disturbing that military intercession in inner clashes in remote nations has gotten ordinary for the United States. † Vladimir Putin. I feel that it is to some degree because of the traditional â€Å"male† way to deal with morals which centers around â€Å"independence, self-sufficiency, mind, will, attentiveness, progressive system, control, culture, amazing quality, item, parsimony, war, and death,† Jaggar, â€Å"Feminist Ethics,† 1992 One can see that these attributes would take into consideration war to be all the more frequently ethically worthy. A â€Å"feminist† way to deal with morals would concentrate more on â€Å"interdependence, network, association, sharing, feeling, body, trust, nonappearance of chain of importance, nature, characteristic, process, happiness, harmony, and life. †Jaggar, â€Å"Feminist Ethics,† 1992 These attributes take into account an increasingly tranquil world. The Syrian system was blamed for utilizing compound weapons against it’s own kin in september of 2013. The weapon of mass pulverization â€Å"Sarin† may have been utilized, and this activity is against global law. The United states felt committed to step in and rebuff the Syrian Government for this since they felt it was on the planets wellbeing to not release these monstrosities unpunished. It could be increasingly risky to let them pull off it since worldwide law could self-destruct and the world could go to disorder if nothing is done to rebuff wrong conduct. This being stated, what is the best strategy to rebuff the Syrian government in the event that they did infact utilize this weapon? Military activity was discussed, and President Obama was eager to assault if fundamental, yet I feel this is extremely hazardous. On the off chance that the United States were to assault the Syrian government without UN endorsement It could have a similar impact as sitting idle. This is on the grounds that the US would likewise be overstepping worldwide law, and this could likewise make the UN self-destruct. In the event that solid nations like the US sidestep UN endorsement on military activities, at that point International law amounts to nothing. â€Å"The world responds by asking: in the event that you can't rely on universal law, at that point you should discover different approaches to guarantee your security. Hence a developing number of nations try to get weapons of mass devastation. This is coherent: in the event that you have the bomb, nobody will contact you. † - Vladimir Putin. Moreover, military activity would not be ethically adequate in this circumstance since it would not be in self preservation; it would be a demonstration of hostility causing more damage than anything else. A strategic methodology would be a positive development for this specific circumstance. This genuine circumstance permits us to perceive how we can pass judgment on the ethical quality of an activity for a bigger scope. Taking everything into account, war can be good, however it is just a system to achieve harmony and wellbeing for a gathering. Every single other alternative ought to be investigated before war can be thought of; this is on the grounds that war includes probably the most unethical activities conceivable; the murdering of others. â€Å"Never imagine that war, regardless of how fundamental, nor how defended, isn't a wrongdoing. †-Ernest Hemingway http://www. brainyquote. com/cites/cites/e/ernesthemi108407. html#zB7XwPTRbCpbv7my. 99 http://www. nytimes. com/2013/09/12/conclusion/putin-supplication for-alert from-russia-on-syria. html? _r=0.

Monday, July 27, 2020

MIT.01 Intro to the Institute, part 1

MIT.01 Intro to the Institute, part 1 I was fifteen and intensely naïve when I graduated from high school and took a gap year. The story of how I decided to take a gap year is probably best saved for another blog post; the important thing is that I did. What can you gain from an entire year free from the bondage of school? Well, the obvious things…twelve months without school is fun, right? Eternal freedom. No homework. Staying up past midnight every night. The dream life. Well, no! No! It’s not fine at all! The first few weeks of this unusual freedom were rife with extravagant eating, curling up in front of my ancient laptop narrowing down my list of unwatched movies and prodding my siblings ever-so-often to end their summer vacation and return to school already. Of course, they eventually did, and for a long time, I was alone. It’s hard to value companionship until you’re starved of it, and my stomach folded into thin layers after most of my high school friends vanished to college, taking on new spheres of life. I grew to resent staying at home, doing nothing, being nothing. Eternal nothingness. I did find a few ways to pass time. I wrote. I wrote a lot. I taught occasionally at my high school, which was a walking distance from where I lived. I tried old math olympiad problems that would have otherwise remained forgotten in dingy, cobwebbed boxes and heard them sigh with relief at renewed importance. I also had a slew of random adventures. I fell into a gutter one evening, scraping my knees and hands. I traveled to Ghana with my family during Christmas. We got to stay near a beach, and watched the waves pound on rocks all day. My cooking skills diversified somewhat, extending from cooking cereal and tea to cooking rice. And somehow,  I decided to apply to MIT. I read my acceptance letter on March 14 at 11:30PM. I was alone in my room, listening to the air-conditioner buzz, my body temporarily robbed of air. I watched the decisions page load before me, my heart thumping, my mind screeching out a jumble of half-hopeful rants and justifications to explain every possible scenario that could play out. Seconds later, words poured out onto the screen. My brain exploded for a moment, and my mouth let out a shriek that burst eardrums and sent my parents hurtling into the room. A month later, I was in the United States for the first time, breathing in the awe of CPW. The sound of happy prefroshes, the view of a majestic dome that sprawled into the sky, the hug from an excited host who repeatedly yelled, “This is awesome!”, the understanding that in every direction I looked stood people with amazing abilities and deeply diverse passions. The Campus Preview Weekend for Admitted Students was an explosive summary of the diversity, passionate energy and power brewing through the very air pockets of MIT, and I was able to breathe it in, atmospheric gases of power I could contemplate, be overwhelmed by, but not fully get used to. Eventually, I made the decision to attend MITâ€"how could I not?, my mind yelledâ€"and began the tortuous visa process with the image of the Institute keeping my morale up. As my gap year drew closer to its end, my level of excitement at the prospect of MIT grew. I felt I was readier for college than I might have been upon graduation months ago. I had a greater appreciation for the value of time and had gained new experiences (examples: a growth from no-cooking-skills to hazardous-but-fairly-acceptable-cooking skills, greater time spent with and more appreciation for my family, a chance to work on a huge writing project that eventually became a published novel). Time seems all too much until it is suddenly not enough, and I realized this the hard way. I distinctly remember the metamorphosis of emotions I got as Orientation drew closer. There was the exhilaration after I got my Student Visa, an understanding that MIT was really not a myth, that I was really going there! Growing nervousness as I realized that yes, yes, I was really going there, this place with those incredible people. A nervousness that became masked, internal terror, a skulking feeling of homesickness and great reluctance to change. But time moved on rudely anyway, paying my feelings no attention. I gave my wonderful family members final, long-lasting hugs and took a flight from Nigeria to the US, with a somewhat annoying tag around my neck that declared “Unaccompanied Minor”. On the plane, I watched an Indian Movie, smiling through it, and once the movie images relapsed into silence, I realized I was in a machine that was roaring above the ocean, pushing me further and further away from my family. I pushed my head against my seat and cried for a little while. I was a lot better after that. Feelings should never be bottled in. Seven action movies and an annoying 4-hour layover later, I was in Virginia where my uncle and his family lived. I spent a week with them, and they made sure it was as fun as possible. We went to an amusement park where I bravely risked death on several rides that stretched to the sky. Below is a picture of me (without my glasses for safety reasons) on one of the roller coasters. As you can see, I’m the poster boy for unfazed, unflinching bravery. The girl next to me is my cousin though, and she’s one heck of a screamer. The week I spent with my uncle and his family brimmed with energy. We went shopping endless times. We hung out in a park one nice day for a picnic, during which I got mercilessly attacked by a bunch of merry kids wielding dangerous-looking water guns. We went to different restaurants, where I tried out new food, from shrimp to broccoli to lasagna to some weird meat-like lump I repeatedly poked with my fork before deciding it wasnt breathing. It was an awesome time, and the atmosphere of glee felt strong enough to last the rest of the week. I thought it would. If only life werent so snide. On the day I was meant to return to MIT, on the day my excitement was supposed to reach some kind of crescendo, the happy atmosphere turned sour. Things took a turn for the worse. Quickly. Very quickly.

Sunday, June 28, 2020

Capital in the Twenty - First Century by Thomas Piketty - 275 Words

Capital in the Twenty - First Century by Thomas Piketty (Book Review Sample) Content: Name:Instructor:Economics:Date:Book ReviewCapital in the Twenty-First Century by Thomas Piketty is a great book, especially in matters concerning business. Its importance stems especially from the detailed statistical information it sheds on various economic parameters. It essentially captures the concept of income inequality in a way that very few other books have done so far, provides detailed explanations of the economic variables, and goes further to explain why these factors matter on the bigger picture. In addressing the topic of inequality, Piketty gives some points that explain the problematic nature of wealth distribution over the last several decades.The book starts by offering a historical perspective of inequality. It asserts that since the mid 20th century, there has been a sharp increase in the income earned by the top earners in the workforce. This has been a chief factor in the subsequent rise in the level of inequality among the different wage groups. The book gives shocking statistics that more than half of the total increment in wages in the United States since the late 1970s went to only the top cream of the people, approximately 1%. When such statistics is related to the present prevailing economic environment, it is hard not to agree with the book.The sharp growth in the levels of inequality, according to the book, have been as a result of not only the availability or lack of capital, but also the rapid increment in the salaries on the high side of the wage structure. It also places a claim on the fact that the massive growth in inequality isnà ¢Ã¢â€š ¬t a blanket observation for all the economically stable nations. In the United States, the book cites the salary increments as a bigger player than the capital factor. The author, however, focuses more on the capital factor as a bigger contributor towards inequality in most western countries, especially in Europe.The book is especially on point with the manner in which it de scribes the contradiction or the paradox of capitalism. The claim of the book is its supposition of the fact that the levels of inequality are unlikely to abate anytime soon. The reasons for this is that the realizable capital returns will over the longer run outweigh the rate of economic growth. In other terms, the magnitude of wealth from the investments made is likely to exceed the rate at which these investors utilize it. The overall supposition of this is that people become more productive than before, and the past and present will always tend to threaten the economic sustenance of the future.The book, however, brings about confusion since it fails to explain satisfactorily the income and...

Friday, May 22, 2020

Health Is Not A Luxury - 1499 Words

Health is not a luxury, instead it is the birth right of every individual to have the best possible health care. But when we say that â€Å"Healthcare† is a basic human right, what do we really mean by that? What is being done to achieve healthcare as a fundamental right of an individual? According to the UN HDI 2014 Statistics report, USA stands at the eighth position in the list of developed countries, quite ahead of Canada, Japan, France, Singapore, yet the condition of healthcare is trailing behind.1 The purpose of this systematic review is to address the key challenges in the current US healthcare system and suggest methods to uplift the healthcare status, by adopting healthcare improvement strategies from different countries in the world. The most crucial challenges faced by the American health care can be classified as: a. High healthcare expenditures: USA spends 16.9% of GDP towards healthcare which is much higher than any other developed country in the world, yet the health care outcomes of USA are not even comparable to the developed nations.2 b. High percentage of uninsured citizens: Although the United States leads the world in the amount of money spent on health care, there are approximately 28.5 million American citizens without insurance.3 The ACA mandate has been instrumental in reducing this number to a great extent there is still a huge gap which needs to be filled. c. Ineffective allocation of resources: Another contrasting problem faced by the US healthcareShow MoreRelatedEconomics of Obesity: Health as a Luxury Good2890 Words   |  12 Pagesï » ¿Q1. Economics of obesity: Health as a luxury good There has been a great deal of public hand-wringing over the obesity epidemic in the United Kingdom and elsewhere. However, obesity does not affect all groups equally. Demographically, the poor are far more likely to be obese. The face of a poor person today is unlikely to be that of a person suffering from malnutrition or under-nutrition, but rather an obese person suffering from eating an abundance of calories derived from low-nutrient sourcesRead MoreFast Fashion Vs. Slow ( Luxury ) Fashion1291 Words   |  6 PagesContemporary Design Issues Stage One - Initial Research Essay Fast Fashion vs. Slow (Luxury) Fashion By Alanna Mitchell In 2014, Jean Paul Gaultier retired from his Ready-To-Wear line. In an interview with The Associated Press he said, Too many clothes kills clothes... Fashion has changed.† (1) This change that Gaultier is referring to is known as â€Å"Fast Fashion†. It is a phenomenon within the fashion industry that â€Å"has made its way into Australian consumers’ wardrobes over the past five years†Read MoreControversy On Luxuries On Prisons1615 Words   |  7 PagesMarina G Burruel Mrs. Martinez English 7-8 (1B) January 20, 2015 Controversy on Luxuries in Prisons In today s prison system of the United States, over 75% of the prisoners have the right to many luxuries while incarcerated instead of getting punished for the crimes committed. In fact, many inmates receive better health care than most people in the United States. Certainly, inmates have more luxuries than the citizens who work hard for them. With that in mind, inmates may consider jail betterRead MoreA Study On The Luxury Hotel Industry1724 Words   |  7 Pagesof the paper. This chapter opens explaining qualitative methodology and why it is the chosen method for the study. Then, after, it details the process of data collection. Qualitative Methodology The research question of this study concerns how the luxury hotel industry can adapt to Chinese conceptualizations and consumer behavior. As the study focuses on culture and values, and how they are represented in Chinese actions and expectations, qualitative methods were determined to be the best processRead MoreSales Tax Essay1086 Words   |  5 Pagesare to be taxed as luxury goods disregards the harm it imposes on American women. Today, in thirty eight states, menstrual products such as pads, tampons and menstrual cups are subject to a luxury tax. (Kaeding) This increases their price, a blow that women should not be forced to accept in a country that promises them liberty. No woman can stop herself from menstrating, therefore no woman should pay a tax for the medical products she requires in her everyday life. The luxury tax on menstrual productsRead MoreHeeral1125 Words   |  5 PagesOberoi Group, founded in 1934, operates 28 hotels and three cruisers in five countries under the luxury ‘Oberoi’ and five-star ‘Trident’ brands. The Group is also engaged in flight catering, airport restaurants, travel and tour services, car rentals, project management and corporate air charters. Oberoi Hotels amp; Resorts is synonymous the world over with providing the right blend of service, luxury and quiet efficiency. Internationally acclaimed for all-round excellence and unparalleled levelsRead MoreTax Incentives in South Africa to Increase Student Enrollment and Promote Economic Growth777 Words   |  3 PagesImposing a tax on domestically produced luxury goods and imposing a higher tax on imported luxury goods will benefit our local producers since it will be to expensive to purchase an imported luxury good. This will support local producers and will increase economic growth. There will be in an increase in demand for locally produced luxury goods and this will create a larger market which will create jobs and decrease the unemployment rate. If we look at the car industry for example , the AutomotiveRead MoreThe Reproductive Justice Issues1156 Words   |  5 Pagesclassify feminine hygiene products as â€Å"optional† or as â€Å"luxury items.† This pape r seeks to further examine how women are directly impacted by this reproductive justice issue by providing a micro-level assessment. Furthermore, an examination of how community agencies are affected by lack of access to feminine hygiene products will be explored in addition to a macro-level assessment. Research has found that this inequity threatens both the health and economic well-being of women in Michigan and thatRead MoreThe Importance Of The Women Tax1029 Words   |  5 Pagesfeminine hygiene products as luxury items. This means not only are these products being unjustly categorized as â€Å"luxury,† but also their prices are being raised due to taxation, which worsens the economic burden for women who require them. This tax has been a topic of debate over the years and has been nicknamed â€Å"the tampon tax† or even â€Å"the women tax.† Overall, state legislators should not tax feminine hygiene products as luxuries, as they are necessities to women’s health, happiness and productivityRead MoreSwot Analysis of Chinese Luxury Cars Industry Essay705 Words   |  3 Pagescase in the Chinese auto market. The urge to splurge on luxury represents a new phase for Chinas car industry, the worlds second largest auto market. As overall sales in China ar e swelling, it is a particular concentration of prestigious brands that care causing the high end of the market to expand twice as fast as the industry average. Opportunities The market demand for cars in China relates to a wide variety of variables: the health of the broader economy, personal income, the price of autos

Friday, May 8, 2020

Diversity At Workplace Diversity Within The Workplace

1.0 Diversity at workplace Diversity is a very extensive subject and can be in any genre or class; it is difficult to categorize because of its complex nature. According to Res (2012), diversity can be represented as a variety of human aspects pertaining to different societies and cultures in the workforce or can be explained tolerating the differences. On the contrary, diversity is a quest of differences among human beings in safe, positive and nurturing surroundings and moves beyond simple tolerance with acceptance and respect (Llopis 2011). It is important that in the twenty first century, the hospitality industry is highly valued by the workforce due to high level of mobility between and across the globe which has impacts on the workplace (Baum et al., 2007). According to Ozbilgin (2008), diversity is one of the top global issues and requires management attention. There are both benefits and challenges in managing diversity. However, Golnaz and Hoa (2002) stated that most organizations agree that diversity is a reality and while managing the same, it can be either an asset or a hindrance towards goals (Gleason 2004). Diversity helps the organization become proactive and flexible to new aspects and scenarios (Pwc 2011). To manage a successful diverse workforce, organisations need to promote self-awareness, openness, clear-headedness, adaptability, candour and egalitarianism. This can be a challenging task due to complex human behaviour and biases (Parkin 2009). 2.0Show MoreRelatedDiversity Within The Workplace : Diversity6655 Words   |  27 PagesDiversity in the Workplace What diversity are we talking about? We have ethnic mixes, cultural differences, racial diversity, and religions across the spectrum, wide age ranges, class mixes, and sexual orientation options. The world is so connected that we are doing business by e-mail and computer transmission all over the globe at all hours of the day and week. We encounter a staggering variety of people every day. We have so many types of people in the workforce today that we must define whatRead MoreThe Diversity Within The Workplace1538 Words   |  7 PagesIt is a known fact that in order for companies to flourish, excellent leadership practices and employees go hand-in-hand. What most companies have failed to acknowledge is how diversity within the workplace, especially in management and executive leadership, does not have a very strong presence. Numerous executives have embraced the fact that their hiring practices are inconsistent with their new motto (we do not discriminate†¦), but fail to reali ze that it is more of an internal issue than it isRead MoreThe Diversity Within The Workplace1540 Words   |  7 PagesIt is a known fact that in order for companies to flourish, excellent leadership practices and employees go hand-in-hand. What most companies have failed to acknowledge is how diversity within the workplace, especially in management and executive leadership, does not have a very strong presence. Numerous executives have embraced the fact that their hiring practices are inconsistent with their new motto (we do not discriminate†¦), but fail to realize that it is more of an internal issue than it isRead MoreThe Diversity Within The Workplace1324 Words   |  6 PagesAs the global reach of each organization grows, their objectives have not changed, however, their look toward leadership has. The increased diversity in the workplace requires superior synchronization by identified employees to meet effectively the mission and vision of the organization. This indiv idual must be capable of meeting the needs of subordinates with their area of expertise with set expectations for meeting organizational goals. While the world analyzed this need, several theories becameRead MoreWorkplace Diversity Within The Workplace865 Words   |  4 Pagesmarket today is workplace diversity. Workplace diversity is defined as all characteristics and experiences that defined each employee as individuals, but it can also be misunderstood as discrimination against employees. Diversity can include race, ethnicity, sex, religion, disability and sexual orientation discrimination. One reason why workplace diversity is important is because when you respect your employees productivity rate rises and many companies do not know that. A diverse workplace targets toRead MoreWorkplace Diversity Within The Workplace1322 Words   |  6 Pagespast decade, diversity in the workplace has become one of the most frequently discussed topics in management groups or circles, while i nterested academically in the actual prospective of workplace diversity has remained limited. To be successful in managing diversity in the workplace first one must understand it from a personal perspective. To be able to derive knowledge and strength from one’s differences shows deep faces in character. A book I read in high school defined diversity as â€Å"the mosaicRead MoreDiversity Within The Workplace : What Is Diversity?1857 Words   |  8 PagesDiversity in the workplace What is diversity? Diversity in the workplace means having employees from a wide range of backgrounds. This can include having employees of different ages, gender, ethnicity, physical ability, sexual orientation, religious belief, work experience, educational background, and so on. Advantages of diversity If you have a diverse workplace in your business, you can benefit from the different talents, experiences and perspectives of your employees. Find out more about the advantagesRead MoreDiversity Within A Diverse Workplace1320 Words   |  6 PagesDiversity is seen as the difference among people. These factors include gender, race, ethnicity, age,sexual orientation,religion,capabilities/disabilities. A Diverse workplace,in the last 30 years, become an important issue to emerge, and it’s effects this can have on a organisation. More and more research has gone into a diverse workplace and effects it can have on an organisation competitive advantage. Although having diverse workplace, brings about opportunities and threats, diversity is complexRead MoreThe Diversity Within The Workplace Essay2257 Words   |  10 Pagesrise to a diverse workforce comprising of different individuals who significantly differ from one another. Diversity in the workplace has been defined as the diff erences and similarities among the employees that make up an organization in terms of age, cultural background, sex, religion (Anon., n.d.). The diversity in the workplace in terms of age is more often referred to as generational diversity. The desire by some to stay and work longer has led to the current workforce capturing many generationsRead MoreGender Diversity Within The Workplace1323 Words   |  6 PagesGender Diversity in Work Balance between men and women in the workplace has been absent from modern corporations. More often than not, employers discriminate against women for being the domestic partner. But society is changing, and we are creating a cultural lag in society, meaning that women have proven their capabilities to be far superior than what is commonly presumed time and time again, but society still fosters beliefs built on traditional gender roles. Women are not always domestic humans;

Wednesday, May 6, 2020

Illegal Immigrants in The United States Essay - 770 Words

For many years, people from all over the world have come to the United States seeking a better life for themselves, their spouse, their children and even their childrens children. However, not every immigrant comes into this country legally and many of these illegal immigrants are poor, uneducated and may be carrying contagious diseases. They may arrive here via human trafficking, smuggling or other methods. There is one thing we know for sure that many of them choose to remain silence when they become victims. The three main reasons they become invisible victims are they are not aware they are victimized, they dont know the right way to ask for help and they are afraid to be fired or even deported. Illegal immigrants†¦show more content†¦For example, an illegal immigrant may be working in a restaurant for over twelve hours a day, but doesnt get paid for overtime. Most importantly, some of these workers dont even know they are victimized, they even think certain level of c orporal punishment from their boss is acceptable, because they used to work that way and they believe its the way it should be. After all, they are earning much more than they used to when they were back in their original countries. Some illegal immigrants may know they are victimized, but dont know the right places or ways to call for help. It is not surprise to see even quite a few legal residents dont know where to seek for help in the complex legal system today. Compare to legal residents, illegal immigrants usually dont equipped with language skills thats enough for them to look for right agencies to help before they have stayed in the country for a certain amount of time. Illegal immigrants usually dont have the time and abilities to look for relevant information from TV, newspaper or even Internet. The circle of people, they are dealing with for the most of time, are usually in similar situations. Also because of they dont want everyone to know about their illegal status, it is very hard for people who can help to find them and help them. Even though they know they are victimized and they know exactly how to get help, they may still remain silence to keep their jobShow MoreRelatedIllegal Immigrants : The United States1392 Words   |  6 PagesJoseph Lema Professor Ferrell English 1470 19 Oct 2017 Illegal Immigration Illegal immigrants are coming into the United States at an alarming rate and it is affecting the United States in negative ways. Immigrants are drawn to America to escape poverty, corrupt government, crime, severe danger, drugs, or persecution in their own countries. Immigrants that come to the United States of America are supposed to apply for a Visa when they first arrive. To get a Visa, the peopleRead MoreIllegal Immigrants And The United States1303 Words   |  6 PagesAs history has shown, millions of immigrants have flocked to the United States in attempts to find a better life. The complexity of immigration is much greater now than it was in the early 1900’s. Most immigrants arriving on boats to Ellis Isle would have only been denied if they were deemed to be a criminal or with disease. Individuals must now endure an extensive application process to obtain a green card or visa. Without one of these documents, the person is considered to be here illegally. To dayRead MoreIllegal Immigrants And The United States1248 Words   |  5 Pagesa half million unauthorized immigrants in the United States in 2014. The population has remained stable for five years, and currently makes up three and half percent of the nation’s population. In the United States Labor Force, there were eight million unauthorized immigrants either working or looking for work in 2014. Is it ethical to employ illegal immigrants? According to the Pew Research Center, Currently, â€Å"49% of US citizens agree with the statement â€Å"immigrants today strengthen the countryRead MoreIllegal Immigrants And The United States1418 Words   |  6 PagesAn illegal immigrant, who works for their keep in an unknown country, contributes to taxes, stays out of trouble, and just wants a better life in a foreign country, on unknown land should be recognized for their contribution to that particular society. An illegal immigrant is a person who migrates to a different country i n a way that is in violation of the immigrant laws of that country. Immigration has been a divided topic for many years in America- illegal immigrants are sometimes seen as a burdenRead MoreIllegal Immigrants And The United States Essay1594 Words   |  7 Pagesand illegal immigrants originate to the United States. From all around a world, individuals want to arise toward America for an improved existence for them and their families. America is a freedom-oriented country, where everybody has right how to live their life in their own conditions. I myself, I remain immigrant as well. I came to U.S.A. 5 years ago. American culture remains actual diverse somewhat from other cultures. In this country, we all get the liberty to live our life. The United StatesRead MoreIllegal Immigrants And The United States Essay1527 Words   |  7 PagesIllegal immigrants have been present in every country ever since governments have been established. Even in the time when Christopher Columbus first s et foot on North America, there were already Natives living on the continent. The British immigrants that first established the Thirteen Colonies brought disease and famine with them. Many Natives died from the diseases that the British brought with them. If there had been a federal government in place, the settlers would have been considered illegalRead MoreIllegal Immigrants And The United States1256 Words   |  6 PagesAn immigrant is a person who legally comes to a country to take up a permanent residence. An illegal immigrant is a person who does come to a country without following the established legal procedures of the destination country and who resides in that country without the proper identifications for example: visas, or other documents. Illegal immigrants are sometimes referred to as illegal aliens or undocumented workers. Emigration and immigration are two sides of the same basic act of human migrationRead MoreIllegal Immigrants And The United States1490 Words   |   6 PagesThe United States of America constantly boasts about its homeland security system and the 1.6 million dollar budget it grants to immigrant and border control. However, in the past decade the number of illegal immigrants in the US has risen from around 4 million to 11.6 million. Its insane that the United States is spending the extremely generous sum of 1.6 million dollars on border control out of its 3.8 trillion dollar budget, and is seeing no return! Currently, about 3.5 million illegal immigrantsRead MoreIllegal Immigrants And The United States1848 Words   |  8 Pagesof the United States immigration has reliably transformed into a bit of our nation s fabric which began many years earlier. Just to wind up one of the most sizzling subjects in the United States and as of late with its essential center being illegal immigrants. Illegal immigrants are individuals who enters a country without the administration s authorization. In 2008, the Center for Immigration Studies assessed that there are more than 11 million illegal immigrants in the United States which areRead MoreIllegal Immigrants : The United States1174 Words   |  5 PagesEach year about 2.5 million Illegal immigrants attempt to cross the border, only about 100,000 and less make it through. Illegal immigrants come to America for a better life. They have gradually increased over time since the 1800’s. Illegal immigrants can come from all parts of the world, but most them come from spanish speaking countries (South and Central America). Most Americans believe illegal immigrants are a problem to the U.S.. Also, Illegal immigrants are believed to be bringing crime, but

Dr Jekyll and Mr Hyde Free Essays

Jenna Pascarelli Mrs. Armstrong English 12 December 19, 2012 The Duality of Man During the Victorian Period, people did not believe in dualism and thought it was unacceptable. Robert Louis Stevenson brings the possibility of another self in one person to life in his creation of Dr. We will write a custom essay sample on Dr Jekyll and Mr Hyde or any similar topic only for you Order Now Jekyll and Mr. Hyde. The quote â€Å"Man is not truly one, but truly two† (Stevenson 43), can be defined as every soul contains elements of both good and evil but one is always dominant. Both sides of an individual cannot be strong at the same time; therefore one side becomes stronger and takes over one’s body. Dr. Jekyll allows Hyde to dominate his personality and eventually he is unable to control Hyde as time proceeds. The duality of the brain during the nineteenth century explains that the left and right hemispheres each had its own function. The right hemisphere was supposedly dominated in the brains of the insane while the left hemisphere was associated with civilization. â€Å"While Jekyll exhibits left-hemisphere attributes, Hyde embodies right-hemisphere traits† (Stiles 4). Stevenson gives each of the hemispheres a life of its own in both characters. The left-brained Jekyll overpowered his right-brain urges which lead to the creation of the second persona. This secondary persona starts off as the weaker of the two but eventually grows stronger. For some time, Jekyll had reasoned that there were two natures in himself. Over the years Jekyll repressed his more impulsive side because he was unsure how people would react towards this side. Everyone who knew Jekyll thought he was a respected doctor who was well mannered. Little did they know he had an evil alter ego, Hyde, which was hidden by the disguise of Jekyll. Eventually Jekyll decided to come to a conclusion â€Å"Though so profound a double-dealer, I was in no sense a hypocrite; both sides of me were in dead earnest† (Stevenson 42). Jekyll explains that both his sides were equally alike and learns how to deal with each side. Unfortunately, Dr. Jekyll could not have possibly foreseen what problems his separate identities would cause. The coexistence of an evil and a good soul in one body presents many problems that occur in the future. The good form in Dr. Jekyll is soon overpowered by the evil nature of Mr. Hyde. Good and evil can no longer be separated in the body. When Mr. Hyde commits a crime, Dr. Jekyll tries to make up for the evil but the situation is morally uncertain. Dr. Jekyll’s possible innocence becomes more doubtful since the two identities are two halves of the same self. It was Dr. Jekyll’s eagerness to put on a mask and taste life of the evil self that has produced these horrible results in the first place. Trying to kill off the Hyde in himself is not an easy task for Jekyll to attempt. The alter ego is a part of who he is and he cannot destroy that part of him. Dr. Jekyll does not approve of the evil things Mr. Hyde does but he cannot control the dominant Hyde who is becoming stronger. Elaine Showalter states, â€Å"The dominant side of the brain represent[s] the dominant gender, and the other repressed gender† (3). This explains that the evil Hyde completely takes over the good Jekyll side because it was hidden for so long that it needed to be expressed. Hyde’s intention was to be able to live his passions freely and to fulfill the evil inclinations that live in his mind without any moral restraints or limits. At last Jekyll, acknowledging defeat, loses control of his alter ego permanently. Succumbing to his dark side, Jekyll surrenders and chooses death. Jekyll feels this is the best way to stop Hyde from committing anymore evil events. Jekyll would permanently be Hyde forever and he did not want to be framed as a murderer. He felt that ending his life would be more beneficial than as living as Hyde for the rest of his life. Sources Cited Stiles, Anne. â€Å"Robert Louis Stevenson’s Jekyll and Hyde and the Double Brain. † SEL: Studies in English Literature, 1500-1900 46. 4 (Autumn 2006): 879-900. Rpt. in Nineteenth-Century Literature Criticism. Vol. 193. Detroit: Gale, 2008. Literature Resource Center. Web. 18 Dec. 2012. Stevenson, Robert Louis. The Strange Case of Dr. Jekyll and Mr. Hyde. New York: Dover Publications, 1991. Print. How to cite Dr Jekyll and Mr Hyde, Essay examples

Diabetes Improvement after Bariatric Surgery

Question: Describe the mechanisms of type 2 Diabetes improvement after bariatric surger? Answer: Link between obecity and diabetes- Type 2 diabetes is one of most common physico-metabolic disorder which is characterized by high glucose level in blood which caused by insulin resistant by internal metabolic system or lack of insulin. Almost 85%-90% cases are type 2 diabetes among diabetes. So here bariatric surgery which is mainly associated with weight loss can be a significant solution for the people suffering from type 2 diabetes. There are some physico-chemical mechanism of body system are responsible for normalization of blood sugar level .Such as caloric restriction hypothesis, gastrointestinal hormonal changes hypothesis etc. (Mingrone , Castagneto-Gissey,2009) 80%-90% of diabetes affected peoples are diagnosed with obese. It provides enough support to the fact that there is an significant co-relation between diabetes obesity. Overweight give extra stress on the human body in a various way. Infact abnormal weight can be a significant factor to insulin resistance of body system. Overweight is common symptom which is generally seen in the people who are take insulin to maintain body glucose level. (Allen et al. 2013).The mechanism is more the insulin is taken more the glucose is absorbed in the cell, which is later transformed into fat. Thus a chance of gaining abnormal body weight is enhances. To get rid of this complications bariatric surgery are often prescribed by the physicians. There are several mechanism to normalize the blood glucose level after bariatric surgery. (Madsbad , Dirksen , Holst 2014). Various mechanism of reducing Type 2 diabetes after bariatric surgery- Research shows type 2 diabetes is markedly improve after a successful weight loss surgery such as bariatric surgery. One long-term study tracked 400 people diagnosed with type 2 diabetes. After 6 year of performing bariatric surgery on them,62% among them have no sign of diabetes. In this blood glucose normalization process various mechanism are associated. These are as follows- 1)Caloric restriction hypothesis- Patient diagnosed with Type 2 diabetes has shown outstanding improvement in insulin sensitivity as well as in bte-cell function also with minimum optimum calorie diet which is maintained consistently as follow up of bariatric surgery where both subcutaneous and central fat decreases.(Bradley , Magkos , Klein ,2012). A small study shows Insulin secretion low down 13.7 mclU/mL to 6.7 mclU/mL(p0.001) in the optimally controlled diet group compared with a fell from 23.2mclU/mL to 12.8 mclU/mL(p0.001) in the bariatric surgery group, performed by Judith Korner, MD, PhD, of Columbia University. 2) Gastrointestinal hormonal changes hypothesis- The concise blood sugar controlling effect of bariatric procedures has been very effective since recent years. In a concise metabolic analysis by Buchwald et al.,shows that controlling rate of blood sugar in type 2 diabetes followed by bariatric procedure, Roux-en-Y gastric bypass (RYGB) were recorded to be 80%. The various gastrointestinal hormones produced from endocrine cells scattered along the whole GI tract. The latest characterization of intestinal metabolism after the bariatric surgery which is associated with entailing systematical alteration of food pathway nutrients through the GI tract composed with the fast frequent controlling of blood glucose level in type 2 diabetes after this operation. (Navarro-Diaz , Serra , Romero , Bonet , Bayes et al. 2006) Figure-1 Img-a Img-b [In the image-a normal gastrointestinal physiology and anatomical structure is shown. In image-b the physiology after RYGB is displayed.] As secretion of L-cell which is located in small intestine plays a significant physic-chemical operative role in management of blood glucose ,and in order to minimizes the L-cell secretion various follow-up chemical function occurred. Such as reduced secretion of the other hormone, glucose-dependent insulinotropic polypeptide from enteroendocrine K cells,which is mainly located in the proximal portion of the small intestine and suppression of ghreline secretion . These all physico-chemical procedures plays a significant role in maintain the blood glucose. In various hormone of stomach Ghrelin glucagon like peptide-1 has an distinct role in post bariatric surgical procedure for normalization of blood sugar level. ( Lieske , Mehta , Milliner , Rule , Bergstralh , Sarr . 2014). a- Decrease Ghrelin hormone Secretion It is known as Hunger hormone, is neuropeptide produced from ghrelin cell. It mainly regulates hunger as well as responsible in distribution and rate of use of energy. Circulation of ghrelin is predominantly occurred in the stomach. Mainly in the alpha beta-cells islet of langerhans ghrelin found to be produced within the pancreas. Here in order to significant concern, with regards to production of insulin ,GHS-R1 a partly co localizes in the pancreatic islet of human body system with the help of insulin positive beta cell. Acknowledging that ghrelin stimulation is one of the significant responsibility of human beta-cell. In this context it is also concern that decrease in ghrelin secretion directly decreases the blood glucose level. (Rodieux , Giusti , D'Alessio , Suter , Tappy 2008) b- Increase glucagon-like peptide-1 release This hormone basically categorized within the incretins group of hormone, the hormone is named like that, as it enhance the secretion of Insulin. L-cells of small intestine are the major source of this hormone. It enhance the secretion of insulin from the pancreas, followed by the enlargement in the volume of cells in the pancreas. After this pancreas produce insulin and stuck glucagon release. Food is the maise are n stimulus of this particular hormone release. In modern days in the formulation of anti hyperglycemic drug designing to mimic this peptide 1 in the human body system to control the blood sugar level in type 2 diabetes. (Sun , Asnicar , Saha , Chan , Smith . 2006) 3) The hindgut hypothesis- After bariatric surgery weight loss is occurring rapidly. Although these are the significant concern of recent research but till these are not clarified prominently. Two hypotheses are available concerning this. The hindgut hypothesis suggests that the rapid transformation of food to the distal bowel improve glucose transformation followed by minimizing glucose level. 4) The foregut hypothesis- Bariatric surgery significantly responsible for weight loss and metabolic changes which is occur after the surgery is associated with alteration in gut hormone which are associated in the glucose maintaining mechanism. In this particular concern the foregut hypothesis suggest that concerning the promoting of insulin resisting factors are till not prominently identified. (Kashyap, Gatmaitan , Brethauer , Schauer 2010) Conclusion- Two significant conclusions are obtained from above discussion and it is also understood that clarification of multiple field is to be needed. First, Insulin response is increased by the alteration of pathway of nutrients with the help of bariatric surgery. Second Maintaining of optimum calorie balance normalize plasma glucose level. References- 1. Mingrone G, Castagneto-Gissey L. 2009. Mechanisms of early improvement/resolution of type 2 diabetes after bariatric surgery. Diabetes Metab. 35 : 518-232. Allen RE, Hughes TD, Ng JL, Ortiz RD, Ghantous MA, et al. 2013. Mechanisms behind the immediate effects of roux-en-Y gastric bypass surgery on type 2 diabetes. Theor. Biol. Med. Model. 10 : 45,4682-10-453. Madsbad S, Dirksen C, Holst JJ. 2014. Mechanisms of changes in glucose metabolism and bodyweight after bariatric surgery. Lancet Diabetes Endocrinol. 2 : 152-644. Bradley D, Magkos F, Klein S. 2012. Effects of bariatric surgery on glucose homeostasis and type 2 diabetes. Gastroenterology. 143 : 897-9125. Navarro-Diaz M, Serra A, Romero R, Bonet J, Bayes B, et al. 2006. Effect of drastic weight loss after bariatric surgery on renal parameters in extremely obese patients: Long-term follow-up. J. Am. Soc. Nephrol. 17 : S213-76. Lieske JC, Mehta RA, Milliner DS, Rule AD, Bergstralh EJ, Sarr MG. 2014. Kidney stones are common afte r bariatric surgery. Kidney Int.7. Rodieux F, Giusti V, D'Alessio DA, Suter M, Tappy L. 2008. Effects of gastric bypass and gastric banding on glucose kinetics and gut hormone release. Obesity (Silver Spring). 16 : 298-3058. Sun Y, Asnicar M, Saha PK, Chan L, Smith RG. 2006. Ablation of ghrelin improves the diabetic but not obese phenotype of ob/ob mice. Cell. Metab. 3 : 379-869. Kashyap SR, Gatmaitan P, Brethauer S, Schauer P. 2010. Bariatric surgery for type 2 diabetes: Weighing the impact for obese patients. Cleve. Clin. J. Med. 77 : 468-76

Tuesday, April 28, 2020

Plague By Albert Camus Essays - Medicine, Plague, Health

Plague By Albert Camus The novel that I chose to do this report on was, "The Plague", by Albert Camus. It is about a plague that hit the European countries in the middle ages. I chose to describe the literary term of parallelism. Here are some following facts about the story's plot that involve parallelism through the novel. The novel begins at Oran where the plague becomes known. The main character, Dr. Gernard Rieux, is a doctor. In the beginning of the story he finds a dead rat on the floor. Even in those times rats were not found dead on the middle of the floor. This was unusual, but he threw out the rat and forgot about it. Eventually the dead rats began to pile into large masses and burned. Soon after there were some people that got very sick, which made Mr. Rieux very curious. These reports of these ill people and the death of the rats were the beginning of the parallelism for this story. Since Bernard was a doctor he was the first to actually attempt to help one of these sick people. Michael was his first patient in this matter. He was the sickest person that the doctor had ever seen. Michael was pale white and vomited often, he hurt so much from the vomiting that he seemed paralyzed. Mr. Rieux tried to help the man the best that he could, but he ended up dying. Michael was the first person to die of this illness. After his death, many cases of this illness were reported widespread. Again more details of sickness and death, this is the parallelism for this novel. As the reports of sickness and death came to inform Dr. Rieux, he tried to comfort and cure the plagued patients. About ninety percent of the people infected had died. He wanted a stop to this plague. Quickly he linked the rats with the people. He knew that the rats began to get sick before the people did. At this time many people had the plague, except for the Chinese visitors. They never were infected. As the plot moves on death, sickness and the plague are still relevant. He studied their behaviors and everyday tasks and learned that they do something that was never often done in these middle ages. Not many people in these days bathed. The doctor began to notice that the people that bathed never got sick. So he asked all of his, still living patients, to take baths frequently. This proved to be the miracle cure for the people. The doctor asked his other fellow doctors to follow the same practice with their patients. The word was spread and the plague was soon wiped out. So as you can see, the literary term of parallelism was deemed very relevant through the ongoing plot. Death, sickness, and the plague epresented the story's parallelism. Albert Camus made parallelism the main literary term for this novel, given away by the title, "The Plague."

Monday, April 13, 2020

Sample Essay on Qualities to Make Me a Good Medical Assistant

Sample Essay on Qualities to Make Me a Good Medical AssistantIf you are looking for a way to highlight your qualifications as a candidate for a medical assistant position, you should consider a sample essay on qualities to make me a good medical assistant. This type of essay should focus on the career choice that you are looking to make. By writing a short essay on the things that you like about being a medical assistant, you can let prospective employers know just what type of assistance you are and that you have great personal qualities that they will be able to utilize when it comes time to interview you.For example, if you enjoy patient care and putting patients' needs first, you may want to include this in your sample essay on qualities to make me a good medical assistant. This could include things like knowing how to treat ailing patients, or how to prepare your patient for discharge from the hospital. It could also include things like helping your patients through their tests and exams so that they can get the best results and not miss out on important medical treatments. It is possible that these types of skills and abilities will not be a main component of your duties in a medical office.You will want to think about the type of medical assistant career path that you would like to follow. If you would like to choose the option of becoming a part-time medical assistant, then it may be necessary to think about some of the things that you like about this career path. If you are planning on working for a large company, or perhaps a hospital, you may find that these things may be common knowledge among your potential employers.You can start by listing off specific details about the things that you do well. It is important to emphasize to potential employers what specific skills and abilities you possess. As you develop your research for potential employers, you will find that they will get a lot of information regarding your personal characteristics and thos e of the people who are likely to hire you.Writing a sample essay on qualities to make me a good medical assistant will take some time and research. Many of the details that you will want to include may not be something that you are comfortable discussing with your personal manager or boss. However, these personal details should be included because they are so important when it comes time to finalize your interview.By doing a bit of research using various search engines, you will be able to gather a list of prospective employers that will be interested in hiring you. Using this list, you will be able to get a better idea of what type of employer or hospital you will be applying to. This information should be discussed with your current manager or your HR representative so that there is no confusion about your career goals.So if you are looking for a way to highlight your qualifications as a candidate for a medical assistant position, then consider writing a sample essay on qualities to make me a good medical assistant. This will help you hone your ideas about what specific career options that you want to pursue. By sharing these details with your potential employers, you will be able to learn which aspects of your personal life are most important to you.

Thursday, March 19, 2020

Goverment essays

Goverment essays Government, the term applied to the institutions and processes, whether formally established or not, by which individuals and groups within a state or society are controlled and regulated for various purposes, such as common defense, general welfare, or internal peace. Thus government is an agency of the state that embraces the organizations and procedures by which laws are enacted, applied, and adjudicated. The major questions and controversies associated with government are not definitional. Rather, they relate to the origins of government, the best way of classifying the various types of government, and, among other things, the distribution of authority among the institutions of government. They also include questions more philosophical in nature, such as: Why is government necessary? How far should government authority extend over individuals or groups? What is the best form of gove3rnment? A discussion of these interrelated questions is the subject of this article. Virtually all political theorists have regarded government in some form as indispensable for society. The doctrine of anarchism, however, holds that government is unnecessary because individuals possess an instinct for mutual cooperation that obviates the need for any direction or control by government. Many anarchists also view government as an instrument designed to protect private property and perpetuate inequalities of wealth. This aspect of anarchist thought is echoed in Marxist theory, which holds that the state and government will "wither away" once the institution of private property is eliminated. Most students of politics subscribe to a much broader view of government's functions and purposes. In The Federalist, a collection of essays interpreting the U.S. Constitution and promoting its ratification, James Madison points to one of the more traditional and widely accepted reasons for government. The differing opinions, passions, and interests of individua...

Tuesday, March 3, 2020

The Trung Sisters of Ancient Vietnam

The Trung Sisters of Ancient Vietnam Beginning in 111 B.C., Han China sought to impose political and cultural control over northern Vietnam, assigning their own governors to oversee existing local leadership, but unease within the region gave birth to brave Vietnamese fighters like Trung Trac and Trung Nhi, The Trung Sisters, who led a heroic yet failed rebellion against their Chinese conquerors.   The pair, born sometime around the dawn of modern history (1 A.D.), were the daughters of a Vietnamese nobleman and military general in the area near Hanoi, and after the death of Tracs husband, she and her sister raised an army to resist and reclaim freedom for Vietnam, thousands of years before it gained its modern independence. Vietnam Under Chinese Control Despite the relatively loose control of Chinese governors in the region, cultural differences made relations between the Vietnamese and their conquerors tense. In particular, Han China followed the strictly hierarchical and patriarchal system espoused by Confucius (Kong Fuzi) whereas the Vietnamese social structure was based on a  more equal status between the sexes. Unlike those in China, women in Vietnam could serve as judges, soldiers, and even rulers and had equal rights to inherit land and other property. To the Confucian Chinese, it must have been shocking that the Vietnamese resistance movement was led by two women - the Trung Sisters, or Hai Ba Trung - but the made a mistake in 39 A.D. when Trung Tracs husband, a noble named Thi Sach, lodged a protest about increasing  tax rates, and in response, the Chinese governor apparently had him executed. The Chinese would have expected a young widow to go into seclusion and mourn her husband, but Trung Trac rallied supporters and launched a rebellion against foreign rule - along with her younger sister Trung Nhi, the widow raised an army of some 80,000 fighters, many of them women, and drove the Chinese from Vietnam. Queen Trung In the year 40, Trung Trac became the queen of northern Vietnam while Trung Nhi served as a top advisor and possibly co-regent. The Trung sisters ruled over an area that included about sixty-five cities and towns and constructed a new capital at Me-linh, a site long associated with the primordial Hong Bang or Loc Dynasty, which legend holds ruled Vietnam from 2879 to 258 B.C. Chinas Emperor Guangwu, who had reunified his country after the Western Han kingdom fell apart, sent his best general to crush the upstart Vietnamese queens rebellion again a few years later and General Ma Yuan was so pivotal to the emperors successes that Mas daughter became the empress of Guangwus son and heir, Emperor Ming. Ma rode south at the head of a battle-hardened army and the Trung sisters rode out to meet him on elephants, in front of their own troops. For more than a year, the Chinese and Vietnamese armies fought for control of northern Vietnam. Defeat and Subjugation Finally, in 43, General Ma Yuan defeated the Trung sisters and their army. Vietnamese records insist that the queens committed suicide by jumping into a river, once their defeat was inevitable while the Chinese claim that Ma Yuan captured and beheaded them instead. Once the Trung sisters rebellion was put down, Ma Yuan and the Han Chinese clamped down hard on Vietnam. Thousands of the Trungs supporters were executed, and many Chinese soldiers remained in the area to ensure Chinas dominance over the lands around Hanoi. Emperor Guangwu even sent settlers from China to dilute the rebellious Vietnamese - a tactic still used today in Tibet and Xinjiang, keeping China in control of Vietnam until 939. Legacy of the Trung Sisters China succeeded in impressing many aspects of Chinese culture upon the Vietnamese, including the civil service exam system and ideas based on Confucian theory. However, the people of Vietnam refused to forget the heroic Trung sisters, despite nine centuries of foreign rule. Even during the decades-long struggles for Vietnamese independence in the 20th century - first against the French colonists, and then in the Vietnam War against the United States - the story of the Trung sisters inspired ordinary Vietnamese. Indeed, the persistence of pre-Confucian Vietnamese attitudes about women may help to account for a large number of female soldiers who participated in the Vietnam War. To this day, the people of Vietnam perform memorial ceremonies for the sisters every year at a Hanoi temple named for them.

Sunday, February 16, 2020

Psychology Research Paper Example | Topics and Well Written Essays - 750 words - 2

Psychology - Research Paper Example He was devoted in the field of physiology and even won the 1904 Nobel Prize for his work in physiology (Martin, 1988). Even though Pavlov was never a psychologist and never wanted to be associated with psychology, he is one of the greatest contributors to the field of psychology. He came through most of his findings through experiments that he performed on animals such as dogs. One of his greatest contributions to the field of psychology is classical conditioning. This is one of the most useful discoveries in the field of psychology and most psychology students have to pass through it in their early years of study. While studying the digestive system of his dog, he noticed something that was very interesting and that led to the development of classical conditioning. He noticed that whenever his assistant entered the room, his dog would immediately start salivating. In the experiment, they started introducing different items to measure the level of salivation that the dog had. He later went on to note that salivation was an automatic response or reflex process (Prokasy, 1965). From the experiments and their outcomes, Pavlov was able to conclude that salivation was a response that could be learned. He noted that the dogs were responding to different stimuli that were presented to them just before food was delivered. They therefore learnt to connect these things with food. He noted that there was the salivary response which came when the dog was presented the food and the conditioning reflex that came when the dog was expecting food. After several experiments, he noted that there was a conditioned stimulus which was food for the dogs and the response which in this case was salivation. This discovery is still the most important discovery in the field of psychology. Apart from the fact that this discovery led to the formation of behavioral psychology, the discovery is used today in the treatment of some health issues as well as the modification of behavior

Sunday, February 2, 2020

Minimum Wage affecting the hunger and environmental nutrition Research Paper

Minimum Wage affecting the hunger and environmental nutrition - Research Paper Example Malnutrition, that is, lack of necessary ingredients for growth and development of people has thus been of great importance in many journals. Even with individuals with enough not feel hungry can still suffer malnutrition. In this case, the malnutrition is not just on lack of food but improper balanced diet. Food security, variety coupled with their affordability defines an environmental nutrition standard of a country. Dietary behaviors influence the whole community in a country though individual practices at household level. These dietary patterns are influenced by various factors among them the income of a household. The purchasing power of a household is determined by its income, holding any other factor constant (Mabli et al., 2010). The introduction of minimum wage requirement exposes various households to a threat of limited amount of goods and services. An income for a household is divided into various categories of expenses (Green-Lapierre et al.’ 2012). These expenses include among others recurrent bill payment, Personal hygiene items, allocation for transportation and most importantly food purchases. This part of argument is mostly concerned with influence of the idea of minimum wage on food purchase part of the expense (Barre, Mizier-Barre & Macintyre, 2011). A considerable high income obviously allows households to exercise a greater freedom in partitioning between the expenses and thus allowing for wide scope of choices, such as with food selection (Kirkpatrick & Tarasuk, 2010). Prices of different commodities are very vital in any decision regarding consumption. The ability to buy of a person is a function of the price under consideration. This means that household with low income or who due to the urgency of the other categories of expenses cannot afford high-nutrition foods are forced to resort to cheaper, less healthy foods (Mabli et al. 2010). Nutritious food consumption

Saturday, January 25, 2020

A Letter From Prison Accounting Essay

A Letter From Prison Accounting Essay Computer Associates International, Inc. (CA) is a software company which sells software products for business. In the case, according to the Generally Accepted Accounting Principles, revenues for the software licensing should be recognized once a contract was signed, the software was delivered, and payment was reasonably assured. In Computer Associates, when the revenue recognition principles were met, it recognized the whole value of the licensing revenues quarterly. However, from the fourth quarter of the fiscal year 1988 to the second quarter of the fiscal year 2001, Computer Associates has backdated some contracts and allows it accountants to record these contracts in the present financial statements. In accordance with the Generally Accepted Accounting Principles, these contracts should be recognized in the next quarter. The future revenues which have recorded in the present financial statements lead to the higher profits and return on investments currently. What Computer Associ ates does is in the violation of the Generally Accepted Accounting Principles. What Computer Associates haves done is a kind of ethical elitism and ethical parochialism. Ethical elitism means that it is significant to maximize the interest of the top stratum or the elite no matter what to scarify and the ethical parochialism refers to that it is important to protect the interests of the individuals in-group. The executives of Computer Associates take their effort to recognize the revenues against the Generally Accepted Accounting Principles in order to protect the reputation of the company and meet the analysts estimation and make the investors confident to the company. However, the investors and shareholders cannot make a good decision by using the improper financial statements. In the case, as the former senior manager of the Computer Associates, Richards try to defense against the crime which makes him in jail. Richards considers that Computer Associates lacks of the information to justify whether the accounting operation in Computer Associates is legal or not. As to him, it is not a big deal for doing this and it is just a timing issue in the revenue recognition. Nevertheless, the revenue recognition in Computer Associates is against the Generally Accepted Accounting Principles. Computer Associates recorded the future revenue in the current financial statements to make the company seems to be profitable. The main function of the financial statements is to show how the company operates and offer the information about the company to the managers and investors. The financial statements are the tools in helping the managers and investors to make the decision. Computer Associates cloud the investors and shareholders about its accurate sales revenues within the current quarter. By using the inaccurate financial statements, strategies and investment are made incorrectly. From the Exhibit 4 in the case, the percentages that properly recorded revenue was inflated by improperly accelerated revenue in every quarter from 2000 to 20001 were all above 10%. Moreover, there are large differences between the announced EPS and EPS without improperly recognized revenue in every quarter from 2000 to 2001. The announced EPS were much higher than the EPS without improperly recognized revenue. Comparing to the analyst EPS estimated, Computer Associates cannot reach the analyst EPS estimated without improperly recognized revenue which is mentioned before. In order to make the financial statements more profitable, Computer Associates use the improper recognized revenue method to make the company seem more profitable. It is unethical for Computer Associates to use allowed accounting flexibility for its revenue recognition. What is accounting flexibility? Accounting flexibility refers to the process that the accountants use their knowledge of accounting rules and standards to manipulate the figures in the financial statement in order to meet some specific purposes. It is about the transformation of the figures in the financial reports from actual figures to the figure which were needed by the preparers, by taking the advantages of the accounting rules and standards. (Naser, 1993). It can help the management to manage the reported figures to be higher or lower. Examples of accounting standards which can use accounting flexibility are as follow: The revaluation of the assets. While in estimation of the assets current value through the depreciation, the estimations are usually made inside the business. It is subjective for the company to estimate the value of the assets. The management has the opportunity to estimate the value on the side of caution or optimism. When doing the estimation in the value of the assets, it refers to the change of the assets, depreciation expenses and the impairment losses. Therefore, the measurement of the assets and income change. The fair value recognition of the plan asset. According to IASB, fair value is defined as the market-based value. It means that the fair value of the plan asset is on the basis of the market transaction. it can be manipulated. When changing the fair value of the plan asset, changes will be recorded in the financial statements. Q2. Richards mentions how difficult it is to operate in the grey areas of accounting and indicated that he might have benefitted from more guidance from senior management. Critically indicate who in a listed firm is responsible for the content of the final accounting reports. Corporate governance is the processes, structures and information which use for coordinating the relations in the management of the corporation. It guarantees the efficiency and the accountability for the mechanism in the corporation to protect the interests of the shareholders. Good corporate governance can help the company to create good corporation culture. The corporation culture creates through the process of the management practices and values which directly come from corporate governance. As in the company, the responsibilities of the major office holders are as follow: Implementing the strategy of the company to make the company operation in the healthy way. Advising the board about the structures of the company and making sure the quality and the quantity of the staff in the company. Providing the accurate information about the company to the board and making the proper prediction for the company. Preparing the accurate financial statements within the Generally Gccepted Accounting Principles In the company, the management should take the responsibility for the accounting reports which is in accordance with the IFRS adopted in Australia. The management should make sure the financial statements are fairly present the financial position and performances of the company. In addition, management must guarantee the financial statements with the accounting standards and prevent them to being fraud. In IFRS 8, management must consider that the most recent pronouncements of other standard setting bodies that use a similar conceptual framework to develop accounting standards, other accounting literature and accepted industry practices. Even though accountants prepare the financial statements in company, the management determines in what ways the financial position illustrate and whether make changes in the financial statements. In conclusion, the management is responsible for the content of the final accounting reports. In Computer Associates case, as a senior manager, Richards did not take his responsibilities to correct the manipulation of the revenues in the financial statements and applied to the sales-driven culture in Computer Associates. He paid more attention to the sales and the revenues in the company. Therefore, with the support of the management included Richards, the improper revenue recognition method was implemented in the company. As for me, possible alternatives can be taken as follow: Changing the accounting policies. The company can use the legal way to manipulate the revenues. For instance, Computer Associates can change the depreciation calculation and change the allocation of the research and development expenses within the Generally Accepted Accounting Principles to reduce the expenses so that the profits can be higher. Changing the time of the transactions. It is helpful for delaying the expenses and the anticipation of the income, which will avoid fighting against the law and the accounting standard. Changing the terms of manipulation. Within Generally Accepted Accounting Principles, some other terms relative to the profits of the company can be manipulated in legal ways. For example, the calculation of the doubtful debts and allowance for uncollectible accounts. Changing the closing date policy of the sales target. As it is mentioned in the case, the customers use delaying tactics to negotiate with Computer Associates to get the better deal. Large proportions of the contract are booked in the final week of the quarter. That makes Computer Associates hard to recognize these contract in the current period and it makes Computer Associates to backdate the contracts. Changing the closing date policy of the sales target is helpful. Computer Associates can short the period for the sales target. For example, it can be closed monthly so that the contracts can be recognized in time. Q3. Consider managements incentives and choices in their actions. What are Computer Associates motivations to manage earnings and the financial ratios as represented in the accounting statements? Internal motivations As Richards mentions in the letter, the company culture in the Computer Associates is a sales-driven culture. It means that the more you sell, the more commissions you can get. The culture leads the company to be aggressive in operation. The goal of the company is to make profit as it can so that it can maximize the shareholders benefits. In addition, the compensations of the executives are on the basis of the sales. The executives would have high compensations when sales associates have reached the goals. According to a study of Massachusetts Institute of Technology in 1983 by Healy, there is a high possibility for choosing and changing accounting procedures in a sales-driven culture with bonus schemes. It is easily to manipulate the accounting figures to maximize the bonus awards. The study also that it is high incidence of voluntary changes in accounting operations in years following the adoption or modification of the bonus reward plan. In Computer Associates, it is so attractive to get the high quantity of compensations by manipulated the revenues. What is more, in Computer Associates, performance in business is a vital criterion. Non-performance is not acceptable in the business. Performance in non-revenue areas should be paid less attentions. In order to perform well, it is reasonable for Computer Associates to manipulate the improper revenue. External motivations The main reason for Computer Associates to manipulate the revenues is to meet the expectations of the market. According to the study of Kasznik and McNichols, the consequences of not meeting the expectations lead to lower future earnings, lower share price, lower market premium and penalization of the markets. Therefore, for Computer Associates, the motivations in order to meet the expectations can be concluded as follows: Future earnings. It is about the stakeholders. The Computer Associates needs to enhance its reputation in their stakeholders, such as distributors and customers. High earnings in the financial reports make the stakeholders more confident for the company. Therefore, the stakeholders would like to do business with the company. Share prices. According to the study of Amat, Blake and Dowds, the accounting flexibility can help to boost the share prices of the company and make the company appeared to less risks for the investors. From the Exhibit 4, it is obvious to see that the EPS without improperly revenue recognition is much lower than the expectation. That means the market will decrease the share price for Computer Associates because of the low revenues. In order to change the situation, Computer Associates should manipulate the revenues to meet the expectations to maintain or increase the share price so that the confidence of the investors can be enhanced. The analysts. As it is mentioned in the case, investors gain information about investments from the analysts instead of the company. The main method which Analysts gather the information about the company is to analyze the financial reports. If the company fails to meet the expectation of the market, the analysts will doubt about the companys future earnings and the credibility. Computer Associates do not want to make the analysts feel doubtful about the companys development so that it tries to manipulate the revenues to meet the expectation of the markets. Q4. All issues related to revenue eventually affect the calculation and recognition of income. Making specific reference to the Comprehensive Income Project initiated by the International Accounting Standards Board (IASB), carefully outline the concept of income that has been proposed by this project and the major issues highlighted. According to IASB, income refers to the increases in the benefits in the accounting period in the form of increases of the assets or the decreases of the liabilities which lead to the increase in equity. Comprehensive income is the changes in equity in a period of transactions and other events and circumstances from sources which are not owned by someone. All the changes in equity should be included in comprehensive income while the investment by owners and distributions to owners should be excluded. Comprehensive income is the sum of historic transaction income and unrealized fair value of the other items. For the historic transaction income, it refers to the entitys income during an accounting period which relative to the companys operation. In IFRS 13, fair value is the value which can be received when selling the assets or paying to transfer a liability in fairly transaction between knowledgeable and willingness parties. Fair value measurement defines as a market-based measuremen t and it is not an entity-specific measurement. Hard income refers to budgeted income that should be recognized during the operations and soft income is the actual income which recognized after the operation. In 2004, a Joint International Working Group on Performance Reporting was established (IASB 2004a). It is helpful in Comprehensive Income Project to establish the standards of the comprehensive income presentation in financial reports. (IASB 2005b) Comprehensive income requires the entity to present all the items relative to income and expense during the period. One single or two statements are accepted. When the Accounting Handbook 2009 was released, the definition of comprehensive income was published. However, income statement was still useful. Therefore, there is a confusion that as the income statement is useful, it seems the comprehensive income approach is quite inconsistent. The comprehensive income requires all the changes in the revenues and expenses and the disclosure items haven been changed. Q5. Critically review and provide an overview summary of a minimum of at least two (2) academic research papers that asses the price relevance of comprehensive income. Show how this research may have influenced subsequent releases and changes in focus by the IASB. In the study of Biddle and Choi, they focused on the debate about the fundamental definition in accounting, the comprehensive income and the consideration of IASB relative to the question. In order to justify those issues, information content, predictive ability and executive compensation contraction were used to examine. The study drew a conclusion that different definition of income makes different decisions and applications and disclosing separately comprehensive income components is useful for making decision. This study is the first study to examine this kind of issue. Another study which had done by Cahan, Courtenay, Gronewoller and Upton, suggested that, to some extent comprehensive income more value relevant than net income. Nevertheless, when doing the asset revaluation increments and foreign currency translation, the effect of comprehensive income was weak and there was no benefit in reporting the separate components of comprehensive income. In conclusion, as for the authors, in the comprehensive income approach, some information was useless, which lead to the comprehensive income did not really benefit the investors. As in a study of comprehensive income, Hanlon had the similar opinion with Cahan, Courtenay, Gronewoller and Upton. In Hanlons study, he mentioned the value relevance of mandated comprehensive income disclosures and discussed whether to choose reported in comprehensive income basis or reported in net income basis. He found that there is no evidence to support the value relevance which would be affected by the comprehensive income. Thus, he suggested that components of comprehensive income were not really value relevant after the controlling for the net income. From my point of view, after reading three essays above, IASB need to focus more on the uses of comprehensive income which IASB makes the entity to report. There are many differences between the different situations so that it leads to different adoptions. Revenue recognition is quite complex. IASB should do more researches to find out whether to use the comprehensive income approach or not when facing different situations.